Difference between revisions of "1991 AIME Problems/Problem 3"

 
m
Line 1: Line 1:
 
== Problem ==
 
== Problem ==
 +
Expanding <math>(1+0.2)^{1000}_{}</math> by the binomial theorem and doing no further manipulation gives
 +
<center><math>{1000 \choose 0}(0.2)^0+{1000 \choose 1}(0.2)^1+{1000 \choose 2}(0.2)^2+\cdots+{1000 \choose 1000}(0.2)^{1000}</math></center>
 +
<center><math>= A_0 + A_1 + A_2 + \cdots + A_{1000},</math></center>
 +
where <math>A_k = {1000 \choose k}(0.2)^k</math> for <math>k = 0,1,2,\ldots,1000</math>. For which <math>k_{}^{}</math> is <math>A_k^{}</math> the largest?
  
 
== Solution ==
 
== Solution ==
 +
{{solution}}
  
 
== See also ==
 
== See also ==
* [[1991 AIME Problems]]
+
{{AIME box|year=1991|num-b=2|num-a=4}}

Revision as of 02:11, 2 March 2007

Problem

Expanding $(1+0.2)^{1000}_{}$ by the binomial theorem and doing no further manipulation gives

${1000 \choose 0}(0.2)^0+{1000 \choose 1}(0.2)^1+{1000 \choose 2}(0.2)^2+\cdots+{1000 \choose 1000}(0.2)^{1000}$
$= A_0 + A_1 + A_2 + \cdots + A_{1000},$

where $A_k = {1000 \choose k}(0.2)^k$ for $k = 0,1,2,\ldots,1000$. For which $k_{}^{}$ is $A_k^{}$ the largest?

Solution

This problem needs a solution. If you have a solution for it, please help us out by adding it.

See also

1991 AIME (ProblemsAnswer KeyResources)
Preceded by
Problem 2
Followed by
Problem 4
1 2 3 4 5 6 7 8 9 10 11 12 13 14 15
All AIME Problems and Solutions